Dowód zasady maksimum.

Ze względu na specyfikę metody - osobny dział.
foe
Użytkownik
Użytkownik
Posty: 24
Rejestracja: 28 paź 2018, o 14:44
Płeć: Mężczyzna
Lokalizacja: wawa
Podziękował: 1 raz

Dowód zasady maksimum.

Post autor: foe »

Witam, nie mam pojęcia jak to równanie rozwiązać oraz czy dobrze do niego doszedłem, proszę o pomoc!:
Stosując zasade maksimum, pokazać że dla liczb naturalnych k >10 prawdziwa jest nierówność:

\(\displaystyle{ 3^{k} -100 \le k!}\)

Doszedłem do tego, że:

\(\displaystyle{ k! + 3 ^{k+1}-100 \le (k+1)!}\)

Co dalej? jest to dobrze?
Awatar użytkownika
Premislav
Użytkownik
Użytkownik
Posty: 15687
Rejestracja: 17 sie 2012, o 13:12
Płeć: Mężczyzna
Lokalizacja: Warszawa
Podziękował: 196 razy
Pomógł: 5221 razy

Re: Dowód zasady maksimum.

Post autor: Premislav »

A co to jest zasada maksimum w liczbach naturalnych? Ja w odniesieniu do naturalnych znam tylko zasadę minimum, zasada maksimum to mi się kojarzy wyłącznie z analizą zespoloną.

Dowód tej nierówności z zastosowaniem zasady minimum mógłby wyglądać tak:
przypuśćmy nie wprost, że dla pewnego \(\displaystyle{ k\in \NN^+, \ k>10}\) teza nie zachodzi, tj.
\(\displaystyle{ 3^k-100>k!}\).
Rozważmy zbiór \(\displaystyle{ \left\{ k\in \NN^+: k> 10 \wedge 3^k-100>k! \right\}}\)
W myśl naszego założenia nie wprost jest to niepusty podzbiór zbioru \(\displaystyle{ \NN}\), ma więc element najmniejszy, weźmy ten element i oznaczmy go przez \(\displaystyle{ k_0}\). Wiemy więc, że
\(\displaystyle{ k_0>10}\) i że \(\displaystyle{ 3^{k_0}-100>(k_0)!}\).
Ale wówczas
\(\displaystyle{ 3^{k_0-1}-100= \frac{3^{k_0}-100}{3}-\frac{200}{3}>\\> \frac{(k_0)!}{3}-\frac{200}{3}=\\=\frac{k_0\cdot (k_0-1)!-200}{3}=\\=(k_0-1)!+ \frac{(k_0-3)(k_0-1)!-200}{3}>(k_0-1)!}\)
Ostatnia nierówność to jakieś bardzo grube szacowania, np.
\(\displaystyle{ k_0-3>10-3>2}\) oraz \(\displaystyle{ (k_0-1)!>5!>100}\).
Gdyby zatem było \(\displaystyle{ k_0-1>10}\), to mielibyśmy sprzeczność z minimalnością \(\displaystyle{ k_0}\), natomiast bezpośrednim rachunkiem sprawdzamy, że dla \(\displaystyle{ k_0=11}\)
zachodzi nierówność wprost przeciwna.
Właściwie jest to sztuczne podejście, w tym przypadku bardziej naturalna wydaje mi się zwykła indukcja.
ODPOWIEDZ